LSAT and Law School Admissions Forum

Get expert LSAT preparation and law school admissions advice from PowerScore Test Preparation.

 Administrator
PowerScore Staff
  • PowerScore Staff
  • Posts: 8917
  • Joined: Feb 02, 2011
|
#23110
Complete Question Explanation

Method of Reasoning. The correct answer choice is (D)

Tom concludes that since employers complain that high school does not currently give graduates the vocational skills necessary for full-time employment, high school students should have to work part time jobs so that they will acquire skills for the job market.

Mary brings up the practical consideration that jobs are already under-available, and simply requiring students to get jobs will not create jobs.

You are asked to identify the relationship of Mary's response to Tom's argument, so you must focus on the fact that she attacks his plan by bringing up an important consideration that may make his plan unworkable.

Answer choice (A) Mary does not analyze any results of Mark's plan; she simply suggests that the plan is not even possible.

Answer choice (B) Mary might argue that Tom's suggested method of reversing the trend of producing vocationally unskilled graduates is unworkable, but that does not mean that Mary thinks that there is no way to reverse the trend, so this choice is wrong. If you assumed that "trend" referred to the availability of part-time employment, you still should have eliminated this choice. There is no reason to believe that Tom has even considered the relevance of the trend in job availability, let alone that he made a mistaken evaluation of the trend.

Answer choice (C) Mary's new information is inconsistent with the idea that Tom's plan would be possible to apply; however, since Tom never states that his plan would be practical, Mary's new information is not an attack on Tom's explicit (stated) premises.

Answer choice (D): This is the correct answer choice. Mary presents the consideration of part-time job availability, which undercuts Tom's assumption that his plan is workable. You might not consider it clear that Tom makes such an assumption, but it is almost certain that someone arguing from Tom's position would believe that any requirements put upon high-school students for their own good are requirements with which students can comply, so you should grant some leeway.

Answer choice (E) Mary offers no solutions, so this choice is wrong.
 EL16
  • Posts: 45
  • Joined: Jul 06, 2017
|
#37752
Hello,

I got this answer correct, and I understand why the correct answer is correct and why A, C, and E are all wrong. However, I am having trouble understanding what B is saying and why it is incorrect. I read the explanation above, but it still isn't making sense to me.

If the "trend" is the high school graduates lacking vocational skills for work, it seems that Mary agrees that this is an unavoidable trend. However, in the explanation for B above, I'm not sure why it says that Mary might think there is NO way to reverse the trend. If anything, wouldn't the answer in B and Mary's statements imply that maybe she also thinks there is no way to reverse the trend, not that she might think there IS a way to reverse it?

Thanks,
Elana
 Eric Ockert
PowerScore Staff
  • PowerScore Staff
  • Posts: 164
  • Joined: Sep 28, 2011
|
#37775
Hi Elana

For answer (B) to be correct, you have to know two things:

1. Tom thinks there is an avoidable problem.
2. Mary thinks that problem is unavoidable.

As with many Method of Reasoning questions, first try to match the abstract terminology in the answer choice(s) to the specific information in the stimulus. If we play devil's advocate with this answer (trying to argue it is correct) then the "problem" here must be the lack of vocational skills required for FT employment. It does appear that Tom believes this avoidable if we require high school students to get PT work. So there's our first prong.

The real problem comes in on the second prong. Does Mary really think this lack of vocational skills is unavoidable? She certainly feels that we can't avoid the problem by requiring them to work part-time, but does that mean Mary thinks that there is no solution to this problem? I don't think we can infer that much about her beliefs.

What you ultimately find here is a common situation in two author stimuli. Mary is attacking Tom's argument, undercutting his assumption. While this questions the validity of Tom's conclusion, it is not necessarily denying the truth of that conclusion. This is a subtle, yet critical, distinction in argumentation. In other words, Mary is saying, "This problem is not avoidable by the solution you are advocating," but she is NOT saying "this problem is unavoidable." For answer (B) to be correct, Mary would have to claim the latter.

One final note: remember that Method questions are ultimately in the Prove Family. Any correct answer must be PROVABLE based on what you read. It is fairly common to see these answers that are close, but just take things a bit too far.

Hope that helps!
 EL16
  • Posts: 45
  • Joined: Jul 06, 2017
|
#37778
Thank you, Eric! That cleared it up for me.
 stephodigie
  • Posts: 5
  • Joined: Jul 16, 2018
|
#59706
I just cant seem to still understand why A is wrong, could yall explain in more detail? Thanks
 Malila Robinson
PowerScore Staff
  • PowerScore Staff
  • Posts: 296
  • Joined: Feb 01, 2018
|
#59755
Hi Stephodigie,
Answer A focuses on an analysis of an undesirable result of a course of action. So let's translate that back into the stimulus, leaving aside the analysis for a moment. The course of action is requiring high school students to work part time jobs to fix their lack of vocational skills. The undesirable result is a little harder to pin down, you might be able to argue that the undesirable result is that since there aren't enough part time jobs, students will not be able to fill the requirement nor gain vocational skills that way. But the biggest issue with Answer A is the analysis; there isn't one. Mary simply dismisses Toms argument, stating that his plan won't work.
Hope that helps!
-Malila
 astroworld
  • Posts: 8
  • Joined: Dec 05, 2019
|
#74479
Hello,

Could you explain in more detail for answer choice C?

The answer choice doesn't state anything about Mary attacking Tom's argument but just being inconsistent with his premise.
I understand why D is the correct answer but I need to be 100% sure why C is incorrect.

Thank you
User avatar
 KelseyWoods
PowerScore Staff
  • PowerScore Staff
  • Posts: 1079
  • Joined: Jun 26, 2013
|
#74491
Hi astroworld!

Mary's information is not inconsistent with any of his premises.

Tom's explicitly stated premises are:

1.) Employers complain that high school graduates lack vocational skills required for full-time employment.
2.) These skills are best acquired on the job.

Mary's information that there are two few part-time jobs for students is not inconsistent with either the statement that employers are complaining or that vocational skills are best acquired on the job. Mary's information and Tom's premises could all be true at the same time. Mary's information simply points out that Tom's conclusion (that we should require high school students to work part-time) is not a practical solution to the problem.

Hope this helps!

Best,
Kelsey
User avatar
 cornflakes
  • Posts: 48
  • Joined: Feb 19, 2021
|
#85718
Hi Powerscore,

I was really torn on C and D here, and from reading through the rationales given I am gleaning that C is incorrect on two major points.

1. Mary provides information that is inconsistent or weakens the validity of Tom's conclusion, not either of the premises.
2. Tom never explicitly states that the part time jobs needed to execute his plan are available - its an underlying assumption of his argument, which is what D actually attacks. He implicitly assumes it, he does not explicitly say it. Therefore, the word "explicit" also makes the answer wrong.

I think the first point is easier to use to eliminate C than the second, but I unfortunately did not recognize the discrepancy between attacking the premise and the conclusion, so I had to do it the hard way.
User avatar
 Ryan Twomey
PowerScore Staff
  • PowerScore Staff
  • Posts: 141
  • Joined: Mar 04, 2021
|
#86424
Hey Cornflakes,

I agree that your first consideration is more important to eliminate C but I would go as far as to say it is the only thing you need to eliminate C. Mary did not provide any information that is inconsistent with Tom's premise, so that is enough to eliminate the answer choice. I think your explanation was too complicated for what the answer choice stated.

D is a perfect answer choice because saying that there are not enough jobs undercuts Tom's argument and Tom did technically assume that there were enough jobs available in his argument. Remember that an assumption is technically not even stated.

A correct method answer choice has to accurately describe the stimulus. So make sure that answer choice accurately describes the stimulus.

Hope this helps.

Best,
Ryan

Get the most out of your LSAT Prep Plus subscription.

Analyze and track your performance with our Testing and Analytics Package.